LSAT and Law School Admissions Forum

Get expert LSAT preparation and law school admissions advice from PowerScore Test Preparation.

User avatar
 Dave Killoran
PowerScore Staff
  • PowerScore Staff
  • Posts: 5853
  • Joined: Mar 25, 2011
|
#45461
Complete Question Explanation
(The complete setup for this game can be found here: lsat/viewtopic.php?t=4080)

The correct answer choice is (A)

Answer choice (A) is the correct answer choice.

Answer choice (B) is incorrect because cage W has three animals, which is impossible according to the first rule.

Answer choice (C) is incorrect because Y has five animals, which is impossible according to the first rule.

Answer choice (D) is incorrect because in the scenario presented there are four remaining snakes to be placed, but no more lizards to put with the snakes.

Answer choice (E) is incorrect because cages with gerbils must have hamsters.
 AJH
  • Posts: 15
  • Joined: Nov 20, 2017
|
#43746
Hello,

I did the setup and found the distributions to be2:4:2:6 or 2:4:4:4. With that, you know that w and x will either have one H and one G, or two of each. Similarly, y and z will either have one L and 3 S, or 2 of each. My question is why A is the correct answer choice over C. I narrowed it down to those two and both seem to work. The only thing I can figure is if I missed in something that limits which of the cages can have which distribution. Help please!
 Adam Tyson
PowerScore Staff
  • PowerScore Staff
  • Posts: 5153
  • Joined: Apr 14, 2011
|
#43754
Thanks for the question, AJH! Your setup looks great, and good job on the numerical distributions. Now, follow this important advice that applies to every logic game at some level: think about the numbers! Check answer choice C again and count the animals in those two cages. Do they match your (accurate) numerical distributions, or is one of them off a little bit? You'll find the answer there!

Get the most out of your LSAT Prep Plus subscription.

Analyze and track your performance with our Testing and Analytics Package.